Difference between revisions of "2021 Mock AMC 8 Problems/Problem 1"
Arcticturn (talk | contribs) (→Solutions) |
Arcticturn (talk | contribs) (→Solution 1) |
||
Line 6: | Line 6: | ||
==Solution 1== | ==Solution 1== | ||
+ | Since <math>1-2+3-4+5-6+7-8</math> is <math>-4</math>, so we have <math>-4 + 9 = 5</math> Therefore, our answer is <math>\boxed{\textbf{(B) } 5} \qquad</math> |
Revision as of 15:04, 16 October 2021
Problem 1
What is the value of ?
Solutions
Solution 1
Since is , so we have Therefore, our answer is